What are the eigenvectors for lambda = -1?

Click For Summary
The discussion focuses on finding the eigenvectors for the eigenvalue lambda = -1 of the matrix A = [0 1 1; 1 0 1; 1 1 0]. The eigenvalues identified are 2 and -1, with the eigenvector for lambda = 2 being v_1 = [1;1;1]. For lambda = -1, the equations derived from Av = -v lead to the condition x + y + z = 0, indicating that the eigenvectors can be expressed in the form <x, y, -x - y>. The resulting eigenvectors for lambda = -1 are linear combinations of the vectors <1, 0, -1> and <0, 1, -1>. This provides a clear method for determining the eigenvectors corresponding to the eigenvalue -1.
Anabelle37
Messages
35
Reaction score
0

Homework Statement



find the general solution to x'=Ax; where A is a 3x3matrix: A=[0 1 1; 1 0 1; 1 1 0]

Homework Equations



det(A-lambda*I)=0

The Attempt at a Solution



i found the eigenvalues to be 2, -1, -1.
for lambda=2 i found the corresponding eigenvector to be a 3x1 martrix v_1 = [1;1;1]

For lambda=-1 i am having trouble finding the eigenvectors. my A-lamda*I matrix is now a 3x3 matrix containing all 1's. how do I find the eigenvector?
 
Physics news on Phys.org
Hi Anabelle37! :smile:

(have a lambda: λ :wink:)
Anabelle37 said:
… for lambda=2 i found the corresponding eigenvector to be a 3x1 martrix v_1 = [1;1;1]

So any other eigenvector msut be perpendicular to [1;1;1] …

does that help? :wink:
 
Think about the basic definition of "eigenvalue" and "eigenvector". If -1 is an eigenvalue for this A, there must be a non-zero vector, v, such that Av= -v.

That is
\begin{bmatrix}0 &amp; 1 &amp; 1 \\ 1 &amp; 0 &amp; 1\\ 1 &amp; 1 &amp; 0\end{bmatrix}\begin{bmatrix}x \\ y \\ z\end{bmatrix}= -\begin{bmatrix}x \\ y \\ z\end{bmatrix}

which means we must have y+ z= -x, x+ z= -y, and x+ y= -z. Those are all the same as x+ y+ z= 0 so that z= -x- y. Any vector in the eigenspace corresponding to eigenvalue -1 is of the form <x, y, -x- y>= <x, 0, -x>+ <0 , y , -y>= x<1, 0, -1>+ y<0, 1, -1>.
 
Question: A clock's minute hand has length 4 and its hour hand has length 3. What is the distance between the tips at the moment when it is increasing most rapidly?(Putnam Exam Question) Answer: Making assumption that both the hands moves at constant angular velocities, the answer is ## \sqrt{7} .## But don't you think this assumption is somewhat doubtful and wrong?

Similar threads

  • · Replies 2 ·
Replies
2
Views
2K
  • · Replies 7 ·
Replies
7
Views
2K
Replies
11
Views
2K
Replies
2
Views
1K
Replies
9
Views
2K
  • · Replies 3 ·
Replies
3
Views
2K
  • · Replies 19 ·
Replies
19
Views
4K
  • · Replies 12 ·
Replies
12
Views
3K
  • · Replies 5 ·
Replies
5
Views
2K
  • · Replies 6 ·
Replies
6
Views
2K